Line 2: Line 2:
 
<math> \sum_{n = 1}^{\infty}(n-1)\mu(E_n) \leq \sum_{n = 1}^{\infty}\int_{E_n}|f|\mbox{d}\mu \leq\sum_{n = 1}^{\infty}n\mu(E_n)</math>.
 
<math> \sum_{n = 1}^{\infty}(n-1)\mu(E_n) \leq \sum_{n = 1}^{\infty}\int_{E_n}|f|\mbox{d}\mu \leq\sum_{n = 1}^{\infty}n\mu(E_n)</math>.
  
Then if <math>\sum_{n = 1}^{\infty}n\mu(E_n) < \infty</math>, then <math> \int_X|f|\mbox{d}\mu < \infty, </math>, i.e. <math>f \in L^1</math>.
+
<math>(\Leftarrow)</math>
 +
If <math>\sum_{n = 1}^{\infty}n\mu(E_n) < \infty</math>, then <math> \int_X|f|\mbox{d}\mu < \infty, </math>, i.e. <math>f \in L^1</math>.
  
If <math>f \in L^1</math>, then <math>\sum_{n = 1}^{\infty}(n-1)\mu(E_n) < \infty</math>. Then <math>\sum_{n = 1}^{\infty}n\mu(E_n) = \sum_{n = 1}^{\infty}(n-1)\mu(E_n) + \sum_{n = 1}^{\infty}\mu(E_n) = \sum_{n = 1}^{\infty}(n-1)\mu(E_n) + \mu(X) < \infty </math>, since <math>X</math> is a finite measure space.
+
<math>(\Rightarrow)</math>
 +
If <math>f \in L^1</math>, then <math>\sum_{n = 1}^{\infty}(n-1)\mu(E_n) < \infty</math>. <math>\sum_{n = 1}^{\infty}n\mu(E_n) = \sum_{n = 1}^{\infty}(n-1)\mu(E_n) + \sum_{n = 1}^{\infty}\mu(E_n) = \sum_{n = 1}^{\infty}(n-1)\mu(E_n) + \mu(X) < \infty </math>, since <math>X</math> is a finite measure space.

Latest revision as of 02:59, 10 July 2008

Since $ \int_X|f|\mbox{d}\mu = \sum_{n = 1}^{\infty}\int_{E_n}|f|\mbox{d}\mu $, and $ E_n=\{x \in X : n-1 \leq |f| \leq n\} $, then $ \sum_{n = 1}^{\infty}(n-1)\mu(E_n) \leq \sum_{n = 1}^{\infty}\int_{E_n}|f|\mbox{d}\mu \leq\sum_{n = 1}^{\infty}n\mu(E_n) $.

$ (\Leftarrow) $ If $ \sum_{n = 1}^{\infty}n\mu(E_n) < \infty $, then $ \int_X|f|\mbox{d}\mu < \infty, $, i.e. $ f \in L^1 $.

$ (\Rightarrow) $ If $ f \in L^1 $, then $ \sum_{n = 1}^{\infty}(n-1)\mu(E_n) < \infty $. $ \sum_{n = 1}^{\infty}n\mu(E_n) = \sum_{n = 1}^{\infty}(n-1)\mu(E_n) + \sum_{n = 1}^{\infty}\mu(E_n) = \sum_{n = 1}^{\infty}(n-1)\mu(E_n) + \mu(X) < \infty $, since $ X $ is a finite measure space.

Alumni Liaison

EISL lab graduate

Mu Qiao